• Want to take part in this year's BoS Trials event for Maths and/or Business Studies?
    Click here for details and register now!
  • YOU can help the next generation of students in the community!
    Share your trial papers and notes on our Notes & Resources page

Mathematical Induction (2 Viewers)

shaon0

...
Joined
Mar 26, 2008
Messages
2,029
Location
Guess
Gender
Male
HSC
2009
Prove this by mathematical induction. I tried n=1 but it does not satisfy it.
 

Trebla

Administrator
Administrator
Joined
Feb 16, 2005
Messages
8,299
Gender
Male
HSC
2006
There's a mistake. It should be (3k + 1) rather than (3k - 1)
 

shaon0

...
Joined
Mar 26, 2008
Messages
2,029
Location
Guess
Gender
Male
HSC
2009
Prove by mathematical induction:
2^n>=1+n for n>=1
I think i have the solution but i don't know whether it is correct.
Here it is:
Let n=1.
LHS=2.
RHS=2.
Thus, LHS=RHS. (can't be bothered to put all the signs)
Assume; 2^n>=1+n is true for all n>=1.
Let n=n+1.
LHS=2^n+1
>=2^n.2
>=(n+1)*2 [by assumption]
>=2n+2.
Thus, 2n+2>=n+2
Thus, n>=0.
I don't know how to prove this. Need some help :) thanks.
 

tommykins

i am number -e^i*pi
Joined
Feb 18, 2007
Messages
5,730
Gender
Male
HSC
2008
Assume true for n = k

2^k > 1+k

Prove true for n = k+1

2^(k+1) > k+2
2.2^k > k+2
2(k+1) > k+2
2k + 2 > k+2

2=2 but 2k > k and k > 0

.'. 2k + 2 > k+2

PS. cbf putting equal signs.
 
Last edited:

lolokay

Active Member
Joined
Mar 21, 2008
Messages
1,015
Gender
Undisclosed
HSC
2009
yeah I think you just say
2n >= n+1
2*2n = 2n+1 >= 2n + 2 >= (n+1) +1 [since n>0]

I think what you did is pretty much right
 

shaon0

...
Joined
Mar 26, 2008
Messages
2,029
Location
Guess
Gender
Male
HSC
2009
lolokay said:
yeah I think you just say
2n >= n+1
2*2n = 2n+1 >= 2n + 2 >= (n+1) +1 [since n>0]

I think what you did is pretty much right
Okay. but according to the law of well-ordered pairs. shouldn't nEN(n be an element of Natural numbers)? This also works for all negative numbers.
 

lolokay

Active Member
Joined
Mar 21, 2008
Messages
1,015
Gender
Undisclosed
HSC
2009
not sure that I get what you're asking
the induction above doesn't work for negative numbers, since it is specified that n>0 - and the equation only asks for n>= 1

if you mean why the inequality happens to work for all negative numbers, you could solve that by saying when that when n=0,1 then the equation gives equality
and since 2n is constantly increasing, and at n=0 has a tangent lower than 1 (by differentiating w.r.t n), and at n=1 has a tangent higher than 1, will be >n+1 when n>1, n<0
 

tommykins

i am number -e^i*pi
Joined
Feb 18, 2007
Messages
5,730
Gender
Male
HSC
2008
You're overcmoplicating the question, shaon0.
 

shaon0

...
Joined
Mar 26, 2008
Messages
2,029
Location
Guess
Gender
Male
HSC
2009
lolokay said:
not sure that I get what you're asking
the induction above doesn't work for negative numbers, since it is specified that n>0 - and the equation only asks for n>= 1

if you mean why the inequality happens to work for all negative numbers, you could solve that by saying when that when n=0,1 then the equation gives equality
and since 2n is constantly increasing, and at n=0 has a tangent lower than 1 (by differentiating w.r.t n), and at n=1 has a tangent higher than 1, will be >n+1 when n>1, n<0
Thanks, but i won't overcomplicate my question as tommykins said above.
Thanks for your help :)
 

tommykins

i am number -e^i*pi
Joined
Feb 18, 2007
Messages
5,730
Gender
Male
HSC
2008
shaon0 said:
Okay sorry. I was just wondering.
Sorry, I took it as something where you questioned the actual answer, didn't know it was your own curiosity.

We know that for any n value, it has to be positive as n > 1

If we assume n = k to be true, we're also assuming k>1

Thus, when you do the induction, it holds as 2k>k where k > 0.
 

shaon0

...
Joined
Mar 26, 2008
Messages
2,029
Location
Guess
Gender
Male
HSC
2009
tommykins said:
Sorry, I took it as something where you questioned the actual answer, didn't know it was your own curiosity.

We know that for any n value, it has to be positive as n > 1

If we assume n = k to be true, we're also assuming k>1

Thus, when you do the induction, it holds as 2k>k where k > 0.
Okay.
 

Users Who Are Viewing This Thread (Users: 0, Guests: 2)

Top